mathkyoproの日記

数学や競プロの問題を解説したりします。

1次元井戸型ポテンシャル (無限深さ)

以下の井戸型ポテンシャル
\begin{equation}
V(x) =
\begin{cases}
0 \ \ (0 \le x \le a) \\
\infty \ \ (\mathrm{else})
\end{cases}
\end{equation} での質量 $ m $ の粒子の固有関数・固有エネルギーを求めます。時間発展しないシュレーディンガー方程式は、
\begin{equation}
\left( -\frac{\hbar^2}{2m} \frac{\mathrm{d}^2}{\mathrm{d}x^2} + V(x) \right) \psi (x) = E \psi (x)
\end{equation} です。$\hbar = h / 2 \pi$ はディラック定数です。ここで $h$ はプランク定数です。また $E$ が固有エネルギー、$\psi (x)$ が波動関数です。従って、$ 0 \le x \le a $ において、
\begin{equation}
-\frac{\mathrm{d}^2}{\mathrm{d}x^2} \psi (x) = \frac{2mE}{\hbar^2} \psi (x)
\end{equation} が成立します。上記微分方程式の解は $A, \ B$ を定数として
\begin{equation}
\psi (x) = A \sin \left( \sqrt{\frac{2mE}{\hbar^2}}x \right) + B \cos \left( \sqrt{\frac{2mE}{\hbar^2}}x \right)
\end{equation} となります。井戸の外ではポテンシャルが無限なので波動関数が存在しないことから、境界条件 $\psi (0) = 0$ が成立せねばならず、$B = 0$ が従います。このとき $A = 0$ だと、恒等的に $\psi (x) = 0$ になってしまい規格化できず不適です。従って、$A \neq 0$ です。以上より、
\begin{equation}
\psi (x) = A \sin \left( \sqrt{\frac{2mE}{\hbar^2}}x \right) \ \ (A \neq 0) \tag{1}
\end{equation} となります。もう $1$ つの境界条件 $\psi (a) = 0$ から、
\begin{equation}
\sqrt{\frac{2mE}{\hbar^2}}a = n \pi \ \ (n \ は正整数)
\end{equation} *1 を得ます。整理すると、
\begin{equation}
E = \frac{n^2 \pi^2 \hbar^2}{2ma^2} = \frac{n^2 h^2}{8ma^2} \ \ (n \ は正整数) \tag{2}
\end{equation} を得ます。式 $(1)$ に式 $(2)$ を代入して、
\begin{equation}
\psi (x) = A \sin \left( n \pi \frac{x}{a} \right) \ \ (A \neq 0) \tag{3}
\end{equation} を得ます。さらに、規格化条件
\begin{split}
1 &= \int_0^a \psi^* (x) \psi (x) \mathrm{d}x \\
&= A^*A \int_0^a \sin^2 (k x) \ \ \ \left( k = \frac{n \pi}{a} \ と置いた。 \right) \\
&= A^*A \int_0^a \frac{1 - \cos (2kx)}{2}\mathrm{d}x \ \ \ \left(\because 半角公式 \right) \\
&= A^*A \int_0^a \left[ \frac{1}{2}x - \frac{1}{4k} \sin(2kx) \right]_0^a \\
&= A^*A \frac{a}{2} \ \ \ \left(\because \sin (2ka) = 0 \right)
\end{split} が成り立ちますが、これを満たす $A$ として
\begin{equation}
A = \sqrt{\frac{2}{a}}
\end{equation} が取れます。*2 従って、式 $(3)$ から、
\begin{equation}
\psi (x) = \sqrt{\frac{2}{a}} \sin \left( n \pi \frac{x}{a} \right)
\end{equation} であり、そのときのエネルギーが
\begin{equation}
E = \frac{n^2 \pi^2 \hbar^2}{2ma^2} = \frac{n^2 h^2}{8ma^2} \ \ (n \ は正整数)
\end{equation} です。

最低エネルギーが $0$ ではありませんが、これは、粒子を $0 \le x \le a$ の領域に閉じ込めたので、不確定性原理から、運動量 $p$ が一定以下の値は取れないという事実と対応しています。











*1:左辺は $0$ 以上です。また、$n = 0$ だと、恒等的に $\psi (x) = 0$ となってしまい不適です。よって $n$ は正です。

*2:厳密には位相の自由度があり、$\theta$ を実数として \begin{equation} A = e^{\mathrm{i}\theta} \sqrt{\frac{2}{a}} \end{equation} と書けます。どの位相をとってもエネルギーなどの物理量に影響はありません。ここでは $\theta = 0$ としました。